$(a+b)^2+a^2$ et $(a+b)^2+b^2$
Réponses
-
Si $b = -a$, ça devrait bien se passer.Si tu souhaites une réponse pour $a,b \in \mathbb{N}^*$, chercher à partir des propriétés des triplets pythagoriciens semble être une bonne piste.
-
Merci @Heuristique
Je sous-entendais que $a$ et $b$ étaient naturels. J'aurais dû le préciser.
Cordialement
Paul -
Bonjour
avec un programme Python, il semble qu'il n'y aurait pas de solutions.from math import* for a in range(-10,10): for b in range(-10,10): c=sqrt((a+b)**2+b**2) d=sqrt((a+b)**2+a**2) if c%1==0 and d%1==0: print(a,b)
on obtient une solution uniquement lorsque $a=-b$.
Donc le problème serait de démontrer qu'il n'existe pas de tels couples $(a,b)$ avec $a\neq b$. -
Méfiance : avec un programme de ce genre, on conclurait (à tort) que $x^2-313y^2=1$ n'a pas de solutions alors qu'il y en a une infinité, dont la plus « petite » est donnée par \[x=32\,188\,120\,829\,134\,849\quad {\text{et}}\quad y=1\,819\,380\,158\,564\,160.\]
-
Sauf erreur de ma part, en considérant $a$ et $b$ premiers entre eux (ce qui ne diminue pas la généralité), la piste des triplets pythagoriciens montre rapidement que c'est impossible)Il ne faut pas respirer la compote, ça fait tousser.
J'affirme péremptoirement que toute affirmation péremptoire est fausse -
Cherchons deux naturels $x$ et $y$, non nuls, tels que $(a+b)^2+a^2=x^2 \quad (1)$ et $(a+b)^2+b^2=y^2 \quad (2)$. Si on considère $(1)$ et $(2)$ comme des équations du second degré d'inconnue $a$, leurs discriminants sont $d1=2x^2-b^2 = u^2$ et $d2=y^2-b^2 = v^2$, où $u$ et $v$ sont des entiers.
On a donc $a=(-b+-u)/2=(-b+-v)/2$, donc $u^2=v^2$, d'où $2x^2-b^2=y^2-b^2$, qui est impossible en entiers non nuls.
-
Salut,
Tu es sûr que ça ne serait pas plutôt $\ \ a=\dfrac{-b\pm u}{4}=\dfrac{-b\pm v}{2}\ \ $ ?
Sinon, j'aimerais bien voir la preuve de @Médiat_Suprème : je suis aussi parti sur les triplet Pythagoriciens, mais je tourne en rond (et pas dans le sens de retomber sur la même chose pour faire une descente infinie . . .) -
Merci d'avoir relevé cette grossière erreur. Je me disais bien que c'était un peu trop facile !
-
Bonjour,
sauf erreur, le $v$ de @J.Faizant n'est autre que $a+b$.
$a=(-b+-v)/2$ donne alors $a=a/2$, soit $a=0$.
J'ai bien dit "sauf erreur"!
Cordialement
Paul
Edit: il y a bien une succession d'erreurs: le $a=(-b+-v)/2$ de @Ben314159 est faux. On a en vérité $a=-b+-v$ qui n'a rien d'étonnant puisque $v=a+b$. Il s'en suit que.... $a=a$. -
On suppose par l'absurde qu'il existe $a$ et $b$ vérifiants de tels équations
alors $ \dfrac{(a+b)^2+a^2}{(a+b)^2+b^2}=x^2 $, avec $x\in \mathbb{Q}$.En divisant le terme de gauche par $a^2$ : $ \dfrac{(1+b/a)^2+1}{(1+b/a)^2+(b/a)^2}=x^2 $,et en posant $b/a=y$ l'équation $ \dfrac{(1+y)^2+1}{(1+y)^2+y^2}=x^2$ admet des solutions rationnelles.
En considérant cette équation d'inconnue $y$, on en déduit qu'il existe $z$ tel que,,$\xcancel{3x^2-1=z^2}$
On a plutôt
$x^4-3x^2+1+z^2=0$,[Pourquoi ne pas écrire $\$$ à la place de dollar ? AD] -
$\dfrac{(1+y)^2+1}{(1+y)^2+y^2}=x^2$ admet bien des solutions rationnelles, par exemple $(1,1)$.
-
Si on cherche à regarder l'équation dans ${\mathbb Q}$, alors, en commençant par utiliser les triplets Pythagoriciens (et en supposant $a\!+\!b\!\not=\!0$), on obtient rapidement l'équation $\Big(x\!+\!\dfrac{1}{x}\Big)\Big(y\!-\!\dfrac{1}{y}\Big)=2$ à résoudre dans ${\mathbb Q}_+^*$.
Mais je ne sais pas si ça mène à quelque chose . . . -
Bonsoir,
@Médiat_Suprème aurais-tu la gentillesse de nous donner ta preuve "rapide", éventuellement en utilisant "révéler" si tu crains casser le moral de ceux qui tournent en rond?
Merci d'avance.
Cordialement
Paul
-
Sauf erreur de ma part :
Si $(a+b)^2+a^2$ est un carré alors il existe $p$ et $q$ des entiers de parité différente (sinon $a$ et $b$ ne serait pas premiers entre eux), tels que : :
$a+b = p^2-q^2$
$a = 2 pq$
et donc $b = p^2-q^2 -2pq$
On calcule $(a+b)^2+b^2 = 2(p^4 + q^4 -2(p^2-q^2))$ qui ne peut pas être un carré (sa valuation 2-adique est égale à 1).Il ne faut pas respirer la compote, ça fait tousser.
J'affirme péremptoirement que toute affirmation péremptoire est fausse -
Bonjour,
Merci @Médiat_Suprème
Tu montres que $a$ ne peut être pair. Je suis d'accord!
Mais tu n'étudies pas le cas $a$ impair...
Cordialement
Paul -
J'avais fait les calculs dans l'autre sens de tête et j'avais conclu trop vite que cela donnait à peu près la même chose.Il ne faut pas respirer la compote, ça fait tousser.
J'affirme péremptoirement que toute affirmation péremptoire est fausse -
Le cadeau s'envoleLorsque notre cher Nico, le professeur, intervient dans une question d'analyse, c'est une véritable joie pour les lecteurs..
-
Beau cadeau @gebrane!
Je le déguste demain mais ça sent bon déjà!
En vérité, ma question ne concernait qu'un cas très particulier de la conjecture suivante:
Un quadrilatère $Q$ dont les diagonales sont perpendiculaires, égales et entières n'a pas tous ses côtés entiers.
En l'occurrence, $Q$ est dégénéré: deux de ses côtés sont alignés.
Je crois avoir prouvé un autre cas particulier: celui où $Q$ est un cerf-volant.
Ma preuve est sur le fil "Démonstration de non-existence d'un quintuplet d'entiers, avec condition sur les carrés" initié par @Swingmustard que je salue amicalement. Hélas, personne ne s'est prononcé sur sa validité. Ceci est un appel peu masqué!
Bien à toi
Paul
EDIT: @Lou16, en mp, invalide ma preuve sur le cerf-volant. Voir le fil "Démonstration de non-existence d'un quintuplet d'entiers, avec condition sur les carrés" -
Bonjoura+b=p2−q2
a=2pqDans l'hypothèse où $2pq<p^2-q^2$. Sinon, le point de départ est faux. -
Bonjour,
@PetitLutinMalicieux, j'ai expliqué ci-dessus son erreur à @Médiat_Suprème et il l'a très vite reconnue.
Par ailleurs je souscris à tes remarques générales sur les ceuss qui modifient leurs messages sans le signaler TRES clairement.
Cordialement
Paul -
@PetitLutinMalicieux
Le droit à la rectification est un droit républicain. Pourquoi voudrais-tu priver les gens de ce droit ?Le droit à la rectification est un principe fondamental dans une société démocratique. Il garantit à chaque individu le droit de corriger des informations erronées ou inexactes qu'il donne ou qui le concernent.
Priver les gens de ce droit irait à l'encontre des valeurs démocratiques . Et c'est quoi cette salade assumer ses bêtises ?Lorsque notre cher Nico, le professeur, intervient dans une question d'analyse, c'est une véritable joie pour les lecteurs.. -
Quand les élèves invoqueront leur droit démocratique à la rectification, tout le monde aura 20/20. Et les classement PISA iront bien se faire voir. En tout cas, les méchants professeurs auront bien du mal à expliquer pourquoi ils ont mis 12/20 à une copie où toutes les réponses sont justes (puisque rectifiées).
-
Gebrane,
le problème n’est pas le droit à la rectification, ce n’est pas contesté ici, mais le fait de modifier un message sans laisser trace des coquilles ou sans même le notifier alors que des échanges ont eu lieu à leur sujet. C’est plutôt impoli de mon point de vue, donc incorrect vis à vis du forum lui-même.En effet, certains messages deviennent hors-sujet alors qu’ils l’étaient. -
Bonjour,
@gebrane,
Je suis bien heureux, comme toi, qu'existe ce droit à la rectification.
En revanche, je ne vois pas qu'il aille contre ce droit de demander à qui a commis une erreur dans ses propos d'avoir la gentillesse de le signifier clairement afin que ses lecteurs ne se conduisent pas sur une voie sans intérêt désormais: ce n'est pas une histoire de morale de merde (pardon, mon Dieu, j'ai péché), c'est juste le respect du lecteur. -
Tu mélanges deux problèmes différents. Dans un forum, on se fixe un but d'aider. Celui qui prend l'initiative pour aider peut se tromper et il a le droit de rectifier le tir. Un lecteur peut demander des éclaircissements en cas de non compréhension de la suite des messages.Lorsque notre cher Nico, le professeur, intervient dans une question d'analyse, c'est une véritable joie pour les lecteurs..
-
Mais gebrane, comprends-tu ce que l’on dit exactement.Le plus souvent cela est fait d’ailleurs : l’auteur écrit [j’ai corrigé une coquille ici]. En rouge ou en gras éventuellement.La lecture du fil va mieux ensuite.
-
Bonjour,Qu'est-ce que c'est que ces bêtises ? Il faut absolument rectifier sans laisser de traces pour les dissimuler !Le droit de rectification permet de corriger des données inexactes vous concernant, il y a plein de forums où la modification des messages n'est pas permise, à minima quand il y a eu une réponse et c'est une bonne chose pour éviter ces comportements irrespectueux.Un bon compromis, selon moi, puisqu'ici la modification est autorisée, est de barrer l'erreur pour la corriger pour que la discussion reste lisible et qu'elle n'induise pas un futur lecteur dans l'erreur. Ainsi tout le monde est content.La philosophie nous enseigne à douter de ce qui nous paraît évident. La propagande, au contraire, nous enseigne à accepter pour évident ce dont il serait raisonnable de douter. (Aldous Huxley)
-
Oui. Étrange discussion d’ailleurs. Je ne suis pas pour « voter une loi par référendum sur ce sujet », c’est même un non sujet puisque c’est souvent et majoritairement lisible. Juste un peu de bonne volonté de comprendre un peu des enjeux assez banals.
-
Dom Le petitlutinmalicieux ne me vise pas, j'ai réagis seulement à sa phrase ''assumer ses bêtises''Lorsque notre cher Nico, le professeur, intervient dans une question d'analyse, c'est une véritable joie pour les lecteurs..
-
Ok. Le mot « bêtise » n’est peut-être pas le bon mot pour tous les messages (erreurs, étourderies, coquille, etc.) mais au moins pour quelques uns. Au passage, dans ce contexte « écrire une bêtise » c’est plutôt amusant que honteux, non ?Mais c’est le fond du message qui m’intéresse un peu plus que la forme.Anecdote : je connais un prof des écoles qui dit, quand un élève répond quelque chose d’insolite « Bonne bêtise !!!! » (à la manière enjouée de dire « Bonne réponse !!! ») avec un ton ironique, avec humour bien sûr et bienveillance (la vraie !).Cela amuse ses élèves.
-
Mais Vassilia a ajouté une dose ""touche personnelle"", elle considère que ceux qui corrigent leurs erreurs sans laisser de traces commettent un acte irrespectueuxLorsque notre cher Nico, le professeur, intervient dans une question d'analyse, c'est une véritable joie pour les lecteurs..
-
Les "bêtises" font partie du raisonnement et l'on apprend de ses erreurs comme de celles d'autrui puisque comme le dit Bachelard, la connaissance est une "erreur rectifiée".Pour ma part je considère qu'il n'est ni grave ni honteux de se tromper, c'est un risque inhérent à la pratique vivante des mathématiques (sauf à ne recopier que des corrigés déjà revus et certifiés) et à la lecture, de la discussion il est certain que l'on apprendra également des erreurs corrigées.Mais cela ne veut pas dire que les raisonnements faux valent autant que les valables. Il n'y a pas de gloire non plus à se tromper. Et quitte à avoir de la fierté autant la mettre à savoir reconnaitre que l'on s'est fourvoyé. Bien entendu cela nécessite que ceux qui corrigent n'en profitent pas pour juger.
-
Bonjour
J'ai remis le message du 18 janvier qui aboutit à une fausse conclusion mais le texte barré ne s'applique pas aux formules mathématiques
je pense que c'est plus lisible de cette façons de laisser les anciens messages même s'ils sont faux pour une meilleure lecure. -
Bon, moi je vais effacer mon message qui donne un lien vers une solution de la questionLorsque notre cher Nico, le professeur, intervient dans une question d'analyse, c'est une véritable joie pour les lecteurs..
-
Bonjour,J'ai fini par apercevoir ce que je crois être une résolution de ce problème qui a la forme d'un labyrinthe tracé dans une jungle de "triplets pythagoriciens".$$\mathcal K: =\left \{n^2\mid n\in \Z\right\},\:\:\:\mathcal E :=\Big\{(a,b)\in\Z^2 \mid a+b>0, \:\:(a+b)^2+a^2, (a+b)^2+b^2\in\mathcal K \Big\}.$$On prouve par "l'absurde" que $\:\mathcal E=\emptyset. \: $ On suppose donc que $\mathcal E\neq \emptyset.\:\:$Soit $(a,b)\in \mathcal E $ tel que $a+b$ soit minimal. Alors $a\wedge b =1, \:\:a\equiv b\equiv 1 \mod 2.\quad$ Soit $\boxed{x=a+b},\:\:y =b.\:\:$ Alors:$$ \:x\in \N^*,\quad x\wedge y =1,\quad x \text{ est pair },\:\:y\text { est impair }, \:\:x^2+y^2 \: \text { et } x^2+(x-y)^2 \text { sont des carrés d'entiers. }$$Alors: $\exists p, q,r,s \in \N^* $tels que:$ \qquad p\wedge q =r\wedge s =1, \quad \boxed{x= 2pq =2rs,} \:\:y =p^2 -q^2, \:\:x-y =r^2-s^2.\quad$ Il vient:$p^2-q^2=2pq-r^2+s^2=2pq -r^2+\left(\dfrac{pq}r\right)^2, \quad (r^2-q^2)p^2-2qr^2 p+ r^2(r^2-q^2) =0.\qquad(\mathbf 1) $Le discriminant de ce polynôme du second degré en $p$ est le carré d'un entier: $\:\:q^2r^2-(r^2 -q^2)^2 =t^2, \:\:t\in \N^*. $En posant $\boxed{X= \dfrac {q^2}t, \:\:Y =\dfrac{r^2}t,}\:\: $cela s'écrit:$\:\:3XY-X^2-Y^2=1. \:$ L'intersection de cette conique avec la droite passant par le point $A(1,1)$, de pente $\dfrac uv,( u\in\Z, v\in \N^*, \:u\wedge v =1)),\:$ donne le paramétrage rationnel suivant: $\:X=\dfrac{u^2-2uv+2v^2}{u^2-3uv +v^2},\:\:Y =\dfrac{2u^2-2uv+v^2}{u^2-3uv +v^2}.$Soient $\boxed{A :=u^2-2uv+2v^2,\:\: B :=2u^2 -2uv +v^2}\:$ et $\pi$ un nombre premier, diviseur de $A\wedge B.\:\:$Alors $\:u^2\equiv v^2 \mod \pi, \quad u\equiv -v \mod \pi,\quad A\equiv 5u^2 \equiv 5v^2\equiv 0 \mod \pi, \quad \pi=5,\quad \boxed{A\wedge B =5^m, \:\:m\in \N.}$Edit En fait, $\:m\leqslant 1 \:\: $ mais cela est sans incidence sur ce qui suit.$$\mathcal F: =\Big\{\left((a+b)^2+a^2,(a+b)^2+b^2\right)\mid a,b\in\Z,\: a+b>0,\:\:a\neq b,\:\:a\wedge b =1. \Big\}.\:\:\:$$Notons $w=-v.\:\:A=(u+w)^2+w^2, \:\:B= (u+w)^2+u^2.\:\:$ Quitte à remplacer$(u,w)$ par $-(u,w), \:$ on peut convenir que $u+w\geqslant 0.$Remarquons que:$\:\:|u|\neq |w|.\:\:$(sinon $u=\pm v, \:X=Y, \:q^2=r^2\: $ contredit $(\mathbf 1)).\:\:$ On déduit: $\:(A,B) \in \mathcal F.$Soit $(C,D) \in \mathcal F$ avec $ \:\:C=(c+d)^2+c^2, \:D =(c+d)^2+d^2$ tel que $C\equiv D\equiv 0 \mod 5.\quad$ Alors $c-d \equiv 0 \mod 5.$Notons $c' =\dfrac {3c +2d}5,\:\:d' =\dfrac{-2c-3d}5. \quad $ On vérifie que $\:\: \dfrac C 5= (c'+d')^2+c'^2, \:\:\dfrac D 5 =(c'+d')^2+d'^2,$$ \left(\dfrac C5,\dfrac D5 \right) \in \mathcal F.\:\:$Cela entraîne par récurrence que $(A',B') :=\left(\dfrac A{5^m},\dfrac B{5^m}\right) \in \mathcal F. \:\:$ Il vient alors:$\dfrac {q^2}{r^2} =\dfrac XY =\dfrac {A'}{B'},\:\:A'\wedge B' =1, \quad A',B' \in \mathcal K, \quad A'=(a'+b')^2 +a'^2 , \quad (a',b') \in \mathcal E,\quad A'\leqslant q^2, \:\:\quad 0< a'+b'\leqslant q <x =a+b.$La minimalité de $a+b $ est ainsi contredite.$\quad \mathcal E=\emptyset.$
-
Voici le fil MSE qui a été effacé plus haut.
https://math.stackexchange.com/questions/3859471/can-q-p-and-p-q-p-be-legs-of-a-pythagorean-triple -
La question est très intéressante, Jlapin,
Et j'ai effacé le lien, car c'est un facteur décourageant pour les nouveaux lecteurs qui s'y intéressent. Le concerné dépasse a bien noté le lien, c'est essentiel.
Lorsque on donne un lien , on freine la discussion, as tu vu quelqu'un discuter de la preuve du lien ?Lorsque notre cher Nico, le professeur, intervient dans une question d'analyse, c'est une véritable joie pour les lecteurs.. -
Il y a plus de chances de voir quelqu'un discuter de la preuve du lien si celui-ci est visible que s'il n'y a pas de lien.
-
On verra, le temps nous le dira
Mesdames, messieurs la discussion est réouverte sur la preuve de MSELorsque notre cher Nico, le professeur, intervient dans une question d'analyse, c'est une véritable joie pour les lecteurs.. -
Bonjour,La rédaction de la preuve fournie dans mon précédent message comporte des maladresses et des imprécisions que je juge utile de corriger.$$\mathcal K: =\left \{n^2\mid n\in \Z\right\},\:\:\:\mathcal E :=\Big\{(a,b)\in\Z^2 \mid a+b\neq 0, \:\:(a+b)^2+a^2\in \mathcal K, \:(a+b)^2+b^2\in\mathcal K \Big\}.$$Il s'agit de démontrer par "l'absurde" que $\:\mathcal E=\emptyset. \: $ On suppose donc que $\mathcal E\neq \emptyset.\:\:$Soit $(a,b)\in \mathcal E $ tel que $|a+b|$ soit minimal. Alors $a\wedge b =1, \:\:a\equiv b\equiv 1 \mod 2.\quad$ Soit $\boxed{x=a+b},\:\:y =b.\:\:$ Alors:$$ x\wedge y =1,\quad x \equiv 0,\:\:y \equiv 1 \pmod 2, \quad x^2+y^2\in\mathcal K ,\:\: x^2+(x-y)^2 \in \mathcal K$$C'est le moment d'entonner le refrain de l'hymne aux "triplets pythagoriciens", qui claironne que:$\exists p, q,r,s \in \Z^* $tels que:$ \qquad p\wedge q =r\wedge s =1, \quad \boxed{x= 2pq =2rs,} \:\:y =p^2 -q^2, \:\:x-y =r^2-s^2.\quad$ Il vient:$p^2-q^2=2pq-r^2+s^2=2pq -r^2+\left(\dfrac{pq}r\right)^2, \quad (r^2-q^2)p^2-2qr^2 p+ r^2(r^2-q^2) =0.\qquad(\bigstar) $Le discriminant de ce polynôme du second degré en $p$ est le carré d'un entier: $\:\:q^2r^2-(r^2 -q^2)^2 =t^2, \:\:t\in \N^*. $En posant $\boxed{X= \dfrac {q^2}t, \:\:Y =\dfrac{r^2}t,}\:\: $cela s'écrit:$\:\:3XY-X^2-Y^2=1. \:$L'intersection de la conique $\mathcal C$ ainsi définie et de la droite passant par le point $P(1,1)$, de pente $\dfrac uv,( u,v\in\Z, \:u\wedge v =1)),\:$ donne ce paramétrage rationnel de $ \mathcal C:\:$$\:\boxed{X=\dfrac{(u-v)^2+v^2}{u^2-3uv +v^2},\:\:Y =\dfrac{(u-v)^2+u^2}{u^2-3uv +v^2}.}$Soient $\boxed{A :=(u-v)^2+v^2,\:\: B :=(u-v)^2+u^2}. \:\:$ Prouvons d'abord que $\:A\wedge B =1\text{ ou }5.$Soit $\pi$ un nombre premier, diviseur de $A\wedge B.\quad$Alors $\:u^2-v^2\equiv B-A \equiv 0 \mod \pi.$$u\equiv v \mod \pi$ est impossible, car cela entraîne $0\equiv A\equiv u^2\equiv v^2 \mod \pi \: $ et contredit $u\wedge v =1.\:$ Il s'ensuit que:$ u\equiv -v \mod \pi,\quad A\equiv 5u^2 \equiv 5v^2\equiv 0 \mod \pi, \quad \pi=5,\quad A\wedge B =5^m, \:\:m\in \N.$Si $m>1,\:$alors $\:u^2\equiv v^2 \mod 5^2,\quad u\equiv -v \mod 5^2, \quad A\equiv 5u^2 \equiv 5v^2\equiv 0 \mod 5^2 ,\:$contredit $u\wedge v =1\:\square$D'autre part :$\:\:u-v\neq 0,\:\:u+v\neq 0.\:\:$(sinon $u=\pm v, \:X=Y, \:q^2=r^2\: $ contredit $(\bigstar)).\:\:$$\bullet\:$ Si $\:A\wedge B =1,\:$ alors $\dfrac{q^2}{r^2}=\dfrac XY =\dfrac AB,\quad A,B \in\mathcal K,\quad (u,-v) \in \mathcal E,\quad (u-v)^2+v^2=A\leqslant q^2,\quad0<|u-v|\leqslant |q|<|x| =|a+b|.$La minimalité de $|a+b| $ est contredite.$\bullet \:$ Si $\:A\wedge B =5,\: $ alors$\: u+v \equiv 0 \mod 5.\:\: $ Notons: $\quad c=\dfrac {3v-2u}5,\:\:d =\dfrac{3u-2v}5.\:\:$Alors:$c,d \in \Z,\quad c+d\neq0,\:$ (car $u+v\neq 0)\quad A':= \dfrac A5 =(c+d)^2+c^2, \quad B':=\dfrac B5=(c+d)^2+d^2.$$\dfrac {q^2}{r^2} =\dfrac XY=\dfrac{A'}{B'}, \quad A'\wedge B' =1,\quad A',B' \in\mathcal K,\quad (c,d)\in \mathcal E,\quad (c+d)^2+c^2 =A'\leqslant q^2,\quad 0<|c+d|\leqslant | q|<|x| =|a+b|.$La minimalité de $|a+b| $ est à nouveau mise à mal.$\quad\square$
-
Merci Lou16Lorsque notre cher Nico, le professeur, intervient dans une question d'analyse, c'est une véritable joie pour les lecteurs..
-
Al-Kashi bonjour,
un exercice que tu vas aimer. On met un triangle rectangle dans un carré de coté de longueur $e$ comme dans la figure. Montrer que les mesures a,b,c,d,x,y et z ne peuvent pas être toutes entières.
L'exercice est interdit pour @Lou16 commence par montrer que $ec=ab$
Lorsque notre cher Nico, le professeur, intervient dans une question d'analyse, c'est une véritable joie pour les lecteurs.. -
Bonsoir Gebrane,
J'y réfléchirai lorsque j'aurai un peu de temps. En attendant, je te propose aussi, ainsi qu'aux intervenants de faire le lien entre le problème évoqué par depasse et celui que je propose. Cela permet aussi d'obtenir une démonstration du résultat recherché dans ce fil, différente de celle proposée par Lou16, que je salue au passage, et de celle proposée dans ton lien au-dessus.
Al-Kashi -
Pour le problème géométrique proposé, on se ramène à la question de depasse du filLorsque notre cher Nico, le professeur, intervient dans une question d'analyse, c'est une véritable joie pour les lecteurs..
-
Bonjour
En utilisant le résultat du fil suivant
https://les-mathematiques.net/vanilla/discussion/2332460/parametrage-de-lequation-diophantienne-a-2-b-2-c-2-d-2#latest
on peut obtenir $\begin{aligned} & x_1^2+x_2^2=x_3^2 \\ & x_1^2+x_4^2=x_7^2 \\ & x_5=\left(1+k^2\right) t+\left(1-k^2\right) y \\ & x_2=2 k y \\ & x_3=\left(1-k^2\right) t+\left(1+k^2\right) y \\ & x_4=2 k t\end{aligned}$
$\begin{aligned} & x_1^2+(2 k y)^2=\left[\left(1-k^2\right) t+\left(1+k^2\right) y\right]^2 \\ & \left\{\begin{array}{l}x_1=m^2-n^2 \\ 2 k t=2 m n \\ \left(1-k^2\right) t+\left(1+k^2\right) y=m^2+n^2\end{array}\right. \\ & \left\{\begin{array}{l}x_1=p^2-q^2 \\ 2 k t=2 p q \\ (1+k) t+\left(1-k^2\right) y=p^2+q^2\end{array}\right.\end{aligned}$
$\left\{\begin{array}{l}2 x+2 y=m^2+n^2+p^2+q^2 \\ \left(1+m^m\right) t+(1-r a) y=m^2+m^2 \\ (1-m r) t+(1+r a) y=r^2+q^2\end{array}\right.$
Ce qui permet d'avoir des solutions rationnelles $$ \begin{aligned}
& \left\{\begin{array}{l}x_1=w^2+n \\
x_2=2 x^2 \\
x_3=\left(1-k^2\right) y+\left(1+k^2\right) y \\
x_n=2 k v
\end{array}\right. \\
& x_u=L w V \\
& \left\{\begin{array}{l}
y=\frac{(1+m n)\left(p^2+a^2\right)-(1-m n)\left(m^2+n\right)}{2(m n+n q)} \\
t=\frac{(1+19)\left(m^2+n^2\right)-(1-r 4)\left(p^2+1\right)}{2(m n+r n)}
\end{array}\right. \\
\end{aligned}
$$
[Dans une expression mathématique, si l'on veut aller à la ligne, IL FAUT IMPÉRATIVEMENT utiliser simultanément "Maj + Retour".
"Retour" seul fait planter la compilation $\LaTeX$. AD]
Connectez-vous ou Inscrivez-vous pour répondre.
Bonjour!
Catégories
- 165.1K Toutes les catégories
- 58 Collège/Lycée
- 22.1K Algèbre
- 37.5K Analyse
- 6.3K Arithmétique
- 58 Catégories et structures
- 1.1K Combinatoire et Graphes
- 13 Sciences des données
- 5.1K Concours et Examens
- 20 CultureMath
- 51 Enseignement à distance
- 2.9K Fondements et Logique
- 10.7K Géométrie
- 83 Géométrie différentielle
- 1.1K Histoire des Mathématiques
- 79 Informatique théorique
- 3.9K LaTeX
- 39K Les-mathématiques
- 3.5K Livres, articles, revues, (...)
- 2.7K Logiciels pour les mathématiques
- 24 Mathématiques et finance
- 337 Mathématiques et Physique
- 5K Mathématiques et Société
- 3.3K Pédagogie, enseignement, orientation
- 10.1K Probabilités, théorie de la mesure
- 801 Shtam
- 4.2K Statistiques
- 3.8K Topologie
- 1.4K Vie du Forum et de ses membres